Consider a linear regression situation with a quantitative response variable Y, a quantitative explanatory variable X, and one categorical explanatory variable with 3 levels. Three indicator variables A1, A2 and A3 are defined, with Ak = 1 if the individual is from level k, and O otherwise (for k=1,2,3). To allow different slopes (for the relationship between X and Y) for each level of the categorical variable, which of the following terms need to be included in the model in addition to Bo + B1X + 8? O A1, A2, and A3. O Aj and A2, but not A3. O A X, A2X, and A3X. O AjX and A2 X, but not A3X.

Answers

Answer 1

Include all three indicator variables (A1, A2, and A3) in the model

To allow different slopes in the relationship between X and Y for each level of the categorical variable, the model should include terms A1X and A2X in A1 and A2 respectively. For example, the model can be expressed as:

Bo + B1X + B2A1X + B3A2X + B4A3

where B2 and B3 represent the slope of the relationship between X and Y for people at levels 1 and 2 of the categorical variable, respectively. represent. The value of X remains constant.

Include all three indicator variables (A1, A2, and A3) in the model. This allows to compare the effect of a given level of a categorical variable on the response variable while keeping the value of X constant. For example, if you include only A1 and A2 in your model, you cannot compare the effects of level 3 of the categorical variables on the response.

Read more about this on brainly.com/question/20340896

#SPJ4


Related Questions

a line with a y-intercept of 4 passes through the point (14, -3). it also passes through point (x, -8). what is the x coordinate for that point?: * a) 8 b) 24 c) -24 d) 6 e) -6

Answers

The x-coordinate for the point  (x, -8), if Y-intercept is 4, and the coordinates of the point is (14, -3), is 24, so option B is correct.

What is line?

An object having an endless length and no width, depth, or curvature is called a line. Since lines can exist in two, three, or higher-dimensional environments, they are one-dimensional things.

Given:

Y-intercept = 4,

Coordinates of point =  (14, -3), (x, -8)

Calculate the line equation as shown below,

y = m x + 4

Here, m is the slope,

Put point (14, -3) in the equation,

-3 = m × 14 + 4

m = -7 / 14

m = -1 / 2

Hence, the equation of a line is,

y = -1/2 x + 4

Put point (x, -8),

-8 = -1/2 x + 4,

-12 = -1/2x

x = 24

To know more about the line:

https://brainly.com/question/2696693

#SPJ1

Can someone help please? Picture is already attached. If its wrong please correct it. This isn’t mine by the way, I’m posting it for someone else to double check if it’s right

Answers

Answer:

You've got it right

Step-by-step explanation:

If you want to get stellar communication on the question then maybe include a number line to visually represent why, but otherwise you have the answer right.

a town's population was 7500 at the beginning of the year 2000 and has been decreasing by 3.2 % each year thereafter.

Answers

Answer:

In 2022, the town's population is 2220.

Step-by-step explanation:

It currently the year 2022. Hence, 22 years have passed since 2000.

2022 − 2000 = 22

To solve for the town's population today, first multiply the years passed since 2000 by the percent decrease each year.

[tex]22 \, \textrm{years} \times \dfrac{3.2 \, \%}{\textrm{year}} = 70.4 \, \%[/tex]

Then, subtract that percent of the population in 2000 from the population in 2000.

[tex]7500-(70.4 \,\% \cdot 7500)[/tex]

[tex]= 7500 - 5280[/tex]

[tex]=2220[/tex]

FIND THE COTANGENT! Help me pls.

Answers

Answer: 5/8

Step-by-step explanation:

Solve
16) -x<-x+7(x-2)

Answers

The solution of the given inequality -x < -x + 7 ( x - 2 ) is given by x > 2.

As given in the question,

Inequality is given by :

-x < -x + 7 ( x - 2 )

Open the parenthesis of the given inequality we have,

⇒ -x < -x + 7x - 14

Collect all the like terms on the same side we get,

⇒ - x < 6x  - 14

Subtract 6x from both the side of the inequality we get,

⇒ - x - 6x < 6x - 6x - 14

⇒ -7x < -14

Divide both the side of the inequality by -7 we get,

⇒ -7x/ -7 > -14 / -7

⇒ x > 2

x has all the value greater than 2.

Therefore, for the given inequality the solution is given by x > 2.

Learn more about inequality here

brainly.com/question/28823603

#SPJ4

Y=x^3-4x^2-20x+48 use the rational zero theorem

Answers

The roots of the given polynomial using the rational zero theorem are;  2, -4 and 6.

How to use the rational zero theorem?

We are given the polynomial;

y = x³ - 4x² - 20x + 48

Since all coefficients are integers, we can apply the rational zeros theorem.

The trailing coefficient (the coefficient of the constant term) is 48

Find its factors (with the plus sign and the minus sign): ±1, ±2, ±3, ±4, ±6, ±8, ±12, ±16, ±24, ±48.

These are the possible values for p.

The leading coefficient (the coefficient of the term with the highest degree) is 1.

These are the possible rational roots:

±1, ±2, ±3, ±4, ±6, ±8, ±12, ±16, ±24, ±48.

Checking the possible roots: if a is a root of the polynomial P(x), the remainder from the division of P(x) by x - a should equal 0 (according to the remainder theorem, this means that P(a)=0

Plugging in those values, the only ones that yield P(a) = 0 are; 2, -4 and 6.

Thus, these are the roots of the given polynomial.

Read more about rational zero theorem at; https://brainly.com/question/17003818

#SPJ1

for cos(theta) = -3/5, and pi/2 < theta < pi, find the following trig values

sin theta
tan theta
csc theta
sec theta
cot theta

Answers

For the given range of theta, cos(theta) is negative, which means that theta is in the third or fourth quadrant. In these quadrants, the sine function is positive, the tangent function is negative, and the cotangent function is negative.

Since cos(theta) = -3/5, we can use the Pythagorean identity to find the value of sin(theta):

sin^2(theta) + cos^2(theta) = 1

sin^2(theta) = 1 - cos^2(theta)

sin^2(theta) = 1 - (-3/5)^2

sin^2(theta) = 1 - 9/25

sin^2(theta) = 16/25

sin(theta) = sqrt(16/25) = 4/5

Therefore, the value of sin(theta) is 4/5.

We can also use the identity cot(theta) = 1/tan(theta) to find the value of cot(theta):

cot(theta) = 1/tan(theta)

cot(theta) = 1/(sin(theta)/cos(theta))

cot(theta) = cos(theta)/sin(theta)

cot(theta) = (-3/5)/(4/5)

cot(theta) = -3/4

Therefore, the value of cot(theta) is -3/4.

The values of the other trigonometric functions can be found using the definitions of these functions:

tan(theta) = sin(theta)/cos(theta) = (4/5)/(-3/5) = -4/3

csc(theta) = 1/sin(theta) = 1/(4/5) = 5/4

sec(theta) = 1/cos(theta) = 1/(-3/5) = -5/3

Therefore, the values of the trigonometric functions for the given range of theta are:

sin(theta) = 4/5

tan(theta) = -4/3

csc(theta) = 5/4

sec(theta) = -5/3

cot(theta) = -3/4

PLEASE ASAP 100 POITNS
a) Jill jumped 6 7/8 feet in the long-jump event. Jill’s best friend jumped 6 15/16 feet. How much farther did Jill’s best friend jump? Describe in words the process you used to solve the problem. (2 points)

b) If Jill’s best friend jumped farther than 6.5 feet, then she beat the school record and the seventh graders earn 50 points. If not, the eighth graders earn 50 points. Which grade should be awarded 50 points**? (1 point)

c) Claire and her partner, Grace, are throwing for the javelin event as a team. Claire threw the javelin 42 5/8 feet and Grace threw it 39 3/5 feet. If Claire and Grace combined their distances, what would their total feet thrown be? Show your work. (2 points)

d) If the total distance of Claire and Grace is greater than 83.7 feet, then the seventh graders earn 50 points. If not, then the eighth graders earn 50 points. Which grade should be awarded 50 points**? (1 point).

Answers

Answer:

Step-by-step explanation:

a) To find how much farther Jill's best friend jumped, we need to subtract Jill's distance from her best friend's distance. First, we need to convert both distances to the same unit, in this case inches. 6 7/8 feet is equal to 6 7/8 * 12 inches/foot = 82 7/8 inches. 6 15/16 feet is equal to 6 15/16 * 12 inches/foot = 83 16/16 inches. Now that both distances are in inches, we can subtract them: 83 16/16 inches - 82 7/8 inches = 1 9/16 inches. Therefore, Jill's best friend jumped 1 9/16 inches farther.

b) To determine if Jill's best friend beat the school record, we need to compare her distance to the record distance. The record distance for seventh graders is 6 feet 1 3/5 inches, which is equal to 6 1/5 * 12 inches/foot = 74 inches. Jill's best friend jumped 83 16/16 inches, which is greater than 74 inches. Therefore, Jill's best friend beat the school record and the seventh graders should be awarded 50 points.

c) To find the total distance thrown by Claire and Grace, we need to add their individual distances. First, we need to convert both distances to the same unit, in this case inches. 42 5/8 feet is equal to 42 5/8 * 12 inches/foot = 510 5/8 inches. 39 3/5 feet is equal to 39 3/5 * 12 inches/foot = 472 3/5 inches. Now that both distances are in inches, we can add them: 510 5/8 inches + 472 3/5 inches = 983 8/5 inches. Therefore, the total distance thrown by Claire and Grace is 983 8/5 inches.

d) To determine if the total distance thrown by Claire and Grace is greater than 83.7 feet, we need to compare their distance to the record distance. The record distance for seventh graders is 83.7 feet, which is equal to 83.7 * 12 inches/foot = 1004.4 inches. The total distance thrown by Claire and Grace is 983 8/5 inches, which is less than 1004.4 inches. Therefore, the total distance thrown by Claire and Grace is not greater than the record distance and the eighth graders should be awarded 50 points.

Allie had four and three-fourths tubes of purple paint. She used two and one-fourth tubes. Later, she found three and three-eighths more tubes. How many tubes of purple paint does she have now?


five and thirteen over twenty-four tubes

five and nine over twenty-four tubes

five and seven over eight tubes

five and one over eight tubes

Answers

By subtracting and adding mixed numbers, we can see that at the end she has (5 + 7/8) tubes of paint.

How many tubes of purple paint does she have now?

We know that initially Allie had (4 + 3/4) tubes of purple paint, and she sed (2+ 1/4) tubes.

So at this point we need to take the difference between the two mixed numbers, we will get:

(4 + 3/4) - (2 + 1/4) = (4 - 2) + (3/4 - 1/4)

(4 - 2) + (3/4 - 1/4) = 2 + 2/4

2 + 2/4  = 2 + 1/2

Then she founds (3 + 3/8) more. Now we need to add that:

2 + 1/2 + (3 + 3/8) = (2 + 3) + (1/2 + 3/8)

(2 + 3) + (1/2 + 3/8) = 5 + (4/8 + 3/8)

5 + (4/8 + 3/8) = 5 + 7/8

She has 5 + 7/8 tubes of purple paint.

Learn more about mixed numbers:

https://brainly.com/question/21610929

#SPJ1

The original price of a scarf was $16, During a store closing sale, a shopper saved $12 on the scarf, What percentage discount did she receive? Explain or show your reasoning.

Answers

Answer:

The shopper received a 75% discount

Step-by-step explanation:

ok so first we turn it into an equation

let x=percentage discount

16(x)=12

x=.75

discount=75%

triangle p undergoes a sequence of transformations resulting in tringle q. which sequence of transformation could be used to show that triangle q is similar but not congruent to triangle P

Answers

The transformation could be used to show that triangle Q is similar but not congruent to triangle P is Dilation.

What is transformation?

A transformation is a general term for four specific ways to manipulate the shape and/or position of a point, a line, or geometric figure.

Given that, triangle P undergoes a transformations resulting in triangle Q.

We know, Dilation would not change the shape, just the size; the angle measures would be the same, and the ratio of corresponding sides would be equal to the scale factor used in the dilation. This would give us a similar, but not congruent, figure.

Hence, The transformation could be used to show that triangle Q is similar but not congruent to triangle P is Dilation.

For more references on transformations, click;

https://brainly.com/question/11709244

#SPJ1

Can someone explain what the rules are for division algorithm and what it means?

Answers

Answer:

The division algorithm says when a number is divided by a number gives the quotient

Step-by-step explanation:

A soccer coach surveyed the players to determine the number who preferred selling coupon books, magazine subscriptions, or both for their fundraiser. The results are given in the Venn diagram. To the nearest whole percent, what is the value of a in the relative frequency table for the survey results? a = 27% a = 42% a = 81% a = 88%

Answers

The value of a in the relative frequency table for the survey results a = 27%

From the given Venn diagram, the total number of players =  11 + 3 + 7 + 5 = 26

The number of players preferred magazines books which are not coupon books = 7

The relative frequency for the players who preferred magazines books which are not coupon books (a)=

magazine books but not coupon books/ total player = 7 / 26 = 0.269

In percent,

7/26 x 100 = 26.9 = 27%

Hence, a = 27%

Therefore, the value of a in the relative frequency table for the survey results a = 27%

To learn more about venn diagram refer here

https://brainly.com/question/2099071

#SPJ4

James fenced in his backyard. The perimeter of his fence is 20 feet, and the width of his yard is 2 feet wide. Use the perimeter formula to find the length of his rectangular yard in inches: P = 2L + 2W. (1 foot = 12 inches)

Answers

The length of James's rectangular yard, given a fence perimeter of 20 feet and a width of 2 feet, is 8 feet, equivalent to 96 inches.

How to use the perimeter of a rectangle?

Given: Perimeter (P) = 20 feet, Width (W) = 2 feet,

Conversion factor: 1 foot = 12 inches

Using the formula for perimeter: P = 2L + 2W

Substitute the given values: 20 = 2L + 2(2)

Simplify: 20 = 2L + 4

Subtract 4 from both sides: 16 = 2L

Divide by 2: L = 8 feet

Convert feet to inches: Length (in inches) = 8 feet * 12 inches/foot = 96 inches

So, the length of the rectangular yard is 96 inches.

Learn more about perimeter of a rectangle on:

https://brainly.com/question/24571594

#SPJ1

Final answer:

To find the length of James' rectangular yard in inches, we can use the perimeter formula P = 2L + 2W. Given that the perimeter is 20 feet and the width is 2 feet, we can substitute these values into the formula and solve for the length in inches. The length of James' rectangular yard is 108 inches.

Explanation:

To find the length of James' rectangular yard in inches, we can use the perimeter formula P = 2L + 2W. Given that the perimeter is 20 feet and the width is 2 feet, we can substitute these values into the formula. However, since we need the length in inches, we will convert the feet to inches using the conversion factor of 1 foot = 12 inches.

Let's solve:

Convert the width from feet to inches: 2 feet * 12 inches/foot = 24 inchesSubstitute the values into the formula: 20 feet = 2L + 24 inchesSince the units don't match, we need to convert 20 feet to inches: 20 feet * 12 inches/foot = 240 inchesNow we have the equation: 240 inches = 2L + 24 inchesIsolate L by subtracting 24 inches from both sides: 240 inches - 24 inches = 2LSimplify: 216 inches = 2LDivide both sides by 2: 216 inches / 2 = LSolve: L = 108 inches

The length of James' rectangular yard is 108 inches.

Learn more about rectangular yard here:

https://brainly.com/question/34773081

#SPJ1

Jacob's body metabolizes caffeine at a rate of 13.5% per hour (so the amount of caffeine in Jacob's body decreases by 13.5% each hour) a. If Jacob consumes a cup of coffee with 96 mg of caffeine in it, how long will it take for Jacob's body to metabolize half of the 96 mg of caffeine? hours Preview b. If Jacob consumes an energy drink with 212 mg of caffeine in it, how long will it take for Jacob's body to metabolize half of the 212 mg of caffeine? hours Preview c. If Jacob consumes a cup of coffee with c mg of caffeine in it, how long will it take for Jacob's body to metabolize half of the cmg of caffeine? (Hint: your answer will be a numerical value.) hours Preview Submit

Answers

Jacob's body metabolizes caffeine at a rate of 13.5% per hour.

a)Jacob's body will takes 4.77 ~5 hours to metabolize half of the 96 mg of caffeine.

b) If Jacob consumes an energy drink with 212 mg of caffeine in it, Jacob's body will take 4.77948 ~ 5 hours to metabolize half of the 212 mg of caffeine.

c) Jacob consumes a cup of coffee with c mg of caffeine in it. Jacob's body will take 4.77948~ 5 hours to metabolize half of the c mg of caffeine.

What is decay rate?

The volume will slowly decrease at regular intervals and at a regular rate. This growth reduction is calculated using the exponential decay formula. The general form is y = a(1- r)ᵗ

We have, decay rate r = 13.5 % = 0.135

Initial value , a = 96 mg

plugging the value in formula we get,

y = 96(1 - 0.135)ᵗ --(1)

Now, Half of 96 is 48

so, 48 = 96(0.865)ᵗ --(2)

dividing equation by 96 we get

=> 48/96 = (0.865)ᵗ

taking natrual logarithm both sides,

=> ln(1/2) = ln ((0.865))

=> ln(1/2) = t In (0.865)

=> t = ln(1/2)/In (0.865)

=> t = 4.77948

Since, decay rate is constant so, half never changes for any .

a) 4.779~5 hours, long will it take for Jacob's body to metabolize half of the 96 mg of caffeine.

b) Jacob's body to metabolize half of the 212 mg of caffeine is 4.779~ 5 hours.

c) for Jacob's body to metabolize half of the cmg of caffeine is 4.77948.. So, we have 4.77948 ~ 5 hours is decay constant rate.

To learn more about Decay rate , refer:

https://brainly.com/question/27542728

#SPJ4

For which values of x does each expression make sense?
Square root of x+5
Square root of |x|+1
Square root of (-2x)^2
Square root of (-5x)^3

Answers

The value of x that makes sense for the expression is the domain of the expression
The value of x makes sense for √-2x² is 0
The value of x must not exceed 0 for  √(-2x)³ to make sense

What is Angle Sum Property?

The sum of all angles of a triangle is equal to the angle of a straight line i.e. 180°. If we have a triangle ABC, then the Sum of angles A , B, and angle C will be 180 ° and the value of the exterior angle is equal to the sum of two interior opposite angles.

√-2x²

For the above expression to have a defined value, the expression in the bracket must be positive.

However, x² will always be positive, and -2 will ensure that the expression is always negative, except when x = 0

Hence, the value of x makes sense for √-2x² is 0.

√(-2x)³

For the above expression to have a defined value, the expression in the square root must be positive.

However, the power of 3 will ensure that the expression is always positive only when x is less than or equal to 0

Hence, the value of x must not exceed 0 for √(-2x)³ to make sense.

To know more about the Angle sum property visit,

brainly.com/question/21099419

#SPJ1

Simplify for all questions.

Answers

Answer:

3y; -2a; -5y; 3c; -6

-----------------------------------------

Question 1[tex]-\sqrt{9y^2}=-(-3y)= 3y, \ since\ y < 0[/tex]

Question 2[tex]0.5\sqrt{16a^2} =0.5(-4a)=-2a,\ since\ a < 0[/tex]

Question 3[tex]-5\sqrt{y^2} =-5y, since \ y > 0[/tex]

Question 4[tex]3\sqrt{c^2} =3c,\ since \ c \geq 0[/tex]

Question 5[tex]\sqrt{(3-\sqrt{15})^2 } -\sqrt{(3+\sqrt{15})^2 } =-(3-\sqrt{15}) -(3+\sqrt{15})=-6, since\ 3 < \sqrt{15}[/tex]

A horse owner needs to replace four sections of a coral for the horses. The first section
needs 26 yards of fencing, the second needs 10 3/5, the third, 10 1/5 yards, and the fourth,
8 3/10 yards. How many yards are needed to repair four sections of the fencing?
10

Answers

Answer:

55 1/10 yd

Step-by-step explanation:

26 + 10 + 3/5 + 10 + 1/5 + 8 + 3/10 =

= 54 + 4/5 + 3/10 =

= 54 + (8 + 3)/10 =

= 54 + 11/10 =

= 54 + 10/10 + 1/10 =

= 54 + 1 + 1/10 =

= 55 1/10

Answer:

55[tex]\frac{1}{10}[/tex]

Step-by-step explanation:

Convert all the mixed numbers to fractions with a common denominator (10) then add them up:

26 + 53/5 + 51/5 + 83/10 =

26 + 106/10 + 102/10 + 83/10 =

26 + 291/10 =

26 + 29[tex]\frac{1}{10}[/tex] = 55[tex]\frac{1}{10}[/tex]

Which ordered pair is a solution to the system of inequalities shown on the graph?

A)(3, 3)

B) (1,2)

C)(-3,-2)

D) (0, 1)

Answers

Answer:

C)   (-3,-2)

Step-by-step explanation:

Solutions to the graphed system of inequalities are any points that are contained within the overlapping shaded region.

Plot the points on the given coordinate grid.

The only point that is contained with the overlapping shaded region is:

(-3, -2)

Find axis of symmetry

Answers

Answer:

Step-by-step explanation:

x=-4

Find the constant term in the expansion of (2x+1/x)8

Answers

Answer:

the constant term in this problem is 8

I’m pretty sure it’s 8 !!

consider two medical tests, a and b, for a virus. test a is 90% effective at recognizing the virus when it is present, but has a 5% false positive rate (indicating that the virus is present, when it is not). test b is 95% effective at recognizing the virus, but has a 10% false positive rate. the two tests use independent methods of identifying the virus. the virus is carried by 2% of all people.Say that a person is tested for the virus using only one of the tests, and that test comes back positive for carrying the virus.Which test returning positive is more indicative of someone really carrying the virus? Justify your answer mathematically (i.e. writing down your calculations).

Answers

We observe, that P(V|B)>P(V|A), so the person is more likely to have virus, however is still very small probability (only 15%), so in order to confirm illness, he should make one more test.

The area of mathematics known as probability deals with numerical representations of the likelihood that an event will occur or that a statement is true. An event's probability is a number between 0 and 1, where, roughly speaking, 0 denotes the event's impossibility and 1 denotes certainty.

P(V/A)  is not 0.95. It is opposite:

P(A/V)=0.95

From the text we can also conclude, that

P(A/∼V)=0.1

P(B/V)=0.9

P(B/∼V)=0.05

P(V)=0.01

P(∼V)=0.99

P(V/A) is not 0.95. It is opposite:

P(A/V)=0.95

From the text we can also conclude, that

P(A/∼V)=0.1

P(B/V)=0.9

P(B/∼V)=0.05

P(V)=0.01

P(∼V)=0.99

What we need to calculate and compare is P(V/A) and P(V/B)

P(V∩A)=P(A)⋅P(V/A)⇒P(V/A)=P(V∩A)/P(A)

P(V∩A) means, that person has a virus and it is detected, so

P(V∩A)=P(V)⋅P(A/V)

=0.01⋅0.95

=0.0095

P(A) is sum of two options: "Person has virus and it is detected" and "Person has no virus, but it was mistakenly detected", therefore:

P(A)=P(V)⋅P(A/V)+P(∼V)⋅P(A/∼V)=0.01⋅0.95+0.99⋅0.1=0.1085

Dividing those two numbers we obtain

P(V/A)=0.0095/0.1085=0.08755760368663594

Analogically,

P(V/B)=P(V∩B)/P(B)=P(V)⋅P(B/V)/P(V)⋅P(B/V)+P(∼V)⋅P(B/∼V)

=0.01⋅0.9/0.01⋅0.9+0.99⋅0.1

=0.1538461538461539

We see that P(V|B)>P(V|A), meaning the individual is more likely to have a virus, but is still very unlikely (only 15%), therefore person should perform another test to confirm illness.

To learn more about probability visit: brainly.com/question/11234923

#SPJ4

Find all solutions of the equation 2sinx +√3 = 0

The answer is A+Bkπ and C+Dkπ where k is any integer, 0

Answers

Answer:

Step-by-step explanation:

2sin x=-√3

[tex]sin~x=-\frac{\sqrt{3} }{2} =-sin~\frac{\pi }{3} =sin (\pi +\frac{\pi }{3} ),sin(2\pi -\frac{\pi }{3} )\\=sin(\frac{4\pi }{3} +2k\pi ),sin (\frac{5\pi }{3} +2k\pi )\\=sin(A+Bk\pi ),sin(C+Dk\pi )\\x=A+Bk\pi ,C+Dk\pi \\where~A=\frac{4\pi }{3} ,B=2\\C=\frac{5\pi }{3} ,D=2[/tex]

3 1/2 + 4 3/4 + 5 3/10

Answers

Answer: 13.55 or 13 11/20.

Step-by-step explanation: First, you have to find a common denominator, in this case, it would be 20. 2 can go into 20 10 times, 4 can go into 20 5 times, and 10 can go into 20 2 times. Then, you would NOW have 3 10/20 + 4 15/20 + 5 6/20, to have all the equal denominators. When you add, you would get 12 31/20, but now you have to simplify. Now, you SHOULD get 13 11/20 after simplifying. As a decimal, it would be 13.55 I hoped this helped!

Determine the coordinates of the point shown.
Answer ASAP

A coordinate grid shown. There are increments of 0.5 for each grid line on each of the two axes. A point is located at 3 grid lines to the right and 1 grid line down from the origin.

(4 points)


(1.5, −0.5)

(−0.5, 1.5)

(3, −1)

(1, −1)

Answers

Answer:

(1.5, −0.5)

Step-by-step explanation:

A researcher was interested in comparing the resting pulse rates of people who exercise regularly and people who do not exercise regularly. Independent simple random samples were obtained of 16 people who do not exercise regularly and 12 people who do exercise regularly. The resting pulse rate (in beats per minute) of each person was recorded. The summary statistics are as follows.
Do Not Exercise - Do Exercise
x1 = 73.4 beats/min - x2 = 69.7 beats/min
s1 = 10.3 beats/min - s2 = 8.6 beats/min
n1 = 16 - n2 = 12
test the claim that he difference between the mean pulse rate of people who do not exercise regularly is higher than the mean pulse rate of people who exercise regularly.

Answers

We Reject H₀ if t calculated > t tabulated and hence the mean pulse rate of people who do not exercise regularly is higher than the mean pulse rate of people who exercise regularly

We Reject H₀ if t calculated > t tabulated

But in this case,

0.83 is not greater than 2.056

Therefore, we failed to reject H₀

There is no difference between the mean pulse rate of people who do not exercise and the mean pulse rate of people who do exercise

The Null and Alternate hypothesis is given by

Null hypotheses = H₀: μ₁ = μ₂

Alternate hypotheses = H₁: μ₁ ≠ μ₂

The test statistic is given by

Where  is the sample mean of people who do not exercise regularly.

Where  is the sample mean of people who do exercise regularly.

Where  is the sample standard deviation of people who do not exercise regularly.

Where  is the sample standard deviation of people who do exercise regularly.

Where  is the sample size of people who do not exercise regularly.

Where  is the sample size of people who do exercise regularly.

The given level of significance is

1 - 0.95 = 0.05

The degree of freedom is

df = 16 + 12 - 2 = 26

From the t-table, df = 26 and significance level 0.05,

t = 2.056 (two-tailed)

Conclusion:

We Reject H₀ if t calculated > t tabulated

learn more about of exercise here

https://brainly.com/question/17089447

#SPJ4

what is 13 -90
then 50/90=
then + together both answers and then multiply by 26 and what is the answer =

Answers

Answer: if there is an option for -1,989 then it would be that one

Step-by-step explanation:

13 - 90 would give you -77 and then 50/90 is would give you 0.5555555. -77 + 0.5 would be -76.5 which then you multiply by 26 gives you -1,989.

Work out the value of d in the equality below. 0.0043 = 4.3 × 10d​

Answers

Answer:

d= -3

Step-by-step explanation:

Rearrange variables to the left side of the equation: -4.3x 10d= -0.0043

Divide both sides of the equation by the coefficient of variable

Reduce the fraction

Convert fraction into negative exponential form

Convert both sides of the equation into terms with the same with same base

Simplify using exponent power rule

Based on the given conditions, corresponding exponents are equal

Identify the factors in the expression 3(m + 2).
A. 3 and (m + 2)
B. m and 2
OC. 3 and m
D. 3 and 2

Answers

The factors of the expression 3(m + 2) are 3 and (m + 2) and the correct option is A. 3 and (m + 2).

What is a factor

A factor in an expression of number, variable, term or any other longer expression that is multiplied by another. In mathematics, a number or algebraic expression that divides another number or expression evenly without remainder is also called a factor.

The question have an algebraic expression; 3(m + 2)

considering the four options;

A. 3 and (m + 2)

B. m and 2

C. 3 and m

D. 3 and 2

Only option A. with 3 and (m + 2) can be said to be factors as multiplying them will give 3(m + 2) and can both divide the expression 3(m + 2) evenly without a remainder.

In conclusion, 3 and (m + 2) are factors for the given expression and option A. 3 and (m + 2) is correct.

Learn more about factors here:https://brainly.com/question/25829061

#SPJ1

A systems analyst is testing the feasibility of using a new computer system. He wants to see if the new system uses less processing time than the old system. (Assume the original populations are normally distributed.) A sample of 25 jobs was selected and the processing time for each in seconds was recorded on each of the two systems. The results are as follows:Old System: mean = 27.2 seconds, s = 3.2 seconds, n - 25New System: mean = 24.3 seconds, s = 2.1 seconds, n = 25Difference (Old -New): mean = 2.9 seconds, s = 1.4 seconds, n = 25What is the alternative hypothesis for this problem?a. HA: µD≠0b. HA: µD>0c. HA: µD=0d. HA: µD<0

Answers

The alternative hypothesis for this problem b. HA: µD>0

What is meant by hypothesis?

In math, A hypothesis is an assumption made based on some evidence.

Here we have given that systems analyst is testing the feasibility of using a new computer system. He wants to see if the new system uses less processing time than the old system.

And we need to find the alternative hypothesis for this problem

While we looking into the given question we have identified the following values,

Old System: mean = 27.2 seconds, s = 3.2 seconds, n - 25

New System: mean = 24.3 seconds, s = 2.1 seconds, n = 25

Difference (Old -New): mean = 2.9 seconds, s = 1.4 seconds, n = 25

Basically, while written the hypothesis, we have to follow the order of the following,

=> (Old-New, Larger-Smaller>)

Therefore, the correct option is (B).

To know more about Hypothesis here.

https://brainly.com/question/29576929

#SPJ4

Other Questions
what is the most important duty of the texas railroad commission from the standpoint of the rest of the united states? with blocked top-level domain lists, email from entire countries or regions can be blocked and treated as spam. group of answer choices true false suppose stock prices rise. to offset the resulting change in output the federal reserve could group of answer choices increase the money supply. this increase would also move the price level closer to its value before the rise in stock prices. decrease the money supply. this decrease would also move the price level closer to its value before the rise in stock prices. increase the money supply. however, this increase would move the price level farther from its value before the rise in stock prices. decrease the money supply. however, this decrease would move the price level farther from its value before the rise in stock prices. Why might a common word like whisper be considered an example ofonomatopoeia?OA. The word itself sounds similar to whispering.B. The word whisper can be used to create alliteration.C. The word creates a strong visual image of people whispering.D. The word whisper carries strong cultural associations. the plasma membrane of an excitable cell is more permeable to potassium ions becausemultiple choicecalcium ions block na and cl- channels.there are more leak channels for k than na .protein molecules cannot exit through the cell membrane.of its positive electrical charge.there are more gated channels for k . Select the author's claim for the text "Trains: Then and Now." Ancient Egyptians and Greeks developed the first form of train. Cars have had a negative impact on people traveling by train. People have worked to make train travel faster and more efficient. Train travel has proven to be an inefficient way to get around. What is the primary role of local governments? to represent 1.a (3 points) compute the value of capacitor voltage vc if the switch is in the down position (2) for a long time. measure vc and compare to your computed value. how long is "a long time" for this circuit? 1.b (2 points) compute the value of vc if the switch is in the up position (1) for a long time. use the measured value of the battery voltage. measure vc and compare. which layer of the sun has a temperature of about 4500 degrees kelvin and is reddish-orange in color? group of answer choices photosphere chromosphere corona convective zone radiative zone a teacher figures that final grades in the statistics department are distributed as: a, 25%; b, 25%; c, 40%; d, 5%; f, 5%. at the end of a randomly selected semester, the following number of grades were recorded. find only the test statistics to test the claim that the grade distribution for the department is different than expected. be sure to set up the equation. do not conduct the hypothesis testing. grade a b c d f number 42 36 60 8 14 FILL IN THE BLANK. arendt describes thinking as_____concepts we use all the time without deeply considering their meaning. A student is assessing the correlation between the number of workers in a factory and the number of units produced daily. The table shows the data:Number of workers0 10 20 30 40 50 60 70 80 90Number of units 2 52 102 152 202 252 302 352 402 452(Part A: Is there any correlation between the number of workers in a factory and the number of units produced daily? Justify your answer. (4 points)Part B: Write a function that best fits the data. (3 points)Part C: What do the slope and y-intercept of the plot indicate? (3 points) A literal statement fits thegeneralityrealismsituation True or false :if there is a net force acting on an object it means there is an unbalanced force. if x and t represent distance and time, respectively, the c in x = 1/2ct 2 must suppose that 45% of people own dogs. if you pick two people at random, what is the probability that they both own a dog? give your answer as a decimal (to at least 3 places) or fraction Determine the domain and the range of the function.f = {(2,4),(29,-7),(44, 4), (54, 2)} prance, inc., earned pretax book net income of $1,881,000 in 2021. prance acquired a depreciable asset that year, and first-year tax depreciation exceeded book depreciation by $188,100. prance reported no other temporary or permanent book-tax differences. the pertinent u.s. federal corporate income tax rate is 21%, and prance earned an after-tax rate of return on capital of 4%. enter below prance's 2021 deferred tax benefit or expense and any deferred tax asset or liability. hallucinogenic drugs, such as lsd, function by hallucinogenic drugs, such as lsd, function by mimicking the action of dopamine. stimulating serotonin receptors. stimulating receptors for norepinephrine. increasing the production of gaba. blocking acetylcholine receptors. Algebra 1: Write and solve two-variable linear equations: word problemsRiley plans to join a members-only speaker series for $50. As a member, she will pay just $12.50 for each event she attends.Write an equation that shows how the total cost, y, depends on the number of events Riley attends, x.Do not include dollar signs in the equation.